Đến nội dung

redfox nội dung

Có 96 mục bởi redfox (Tìm giới hạn từ 11-05-2020)



Sắp theo                Sắp xếp  

#692051 Có tồn tại vô hạn số tự nhiên q thỏa mãn $\left[\alpha q^2...

Đã gửi bởi redfox on 01-09-2017 - 17:25 trong Số học

Câu trả lời là có. Viết $\alpha =\left [ x_0;x_1,x_2... \right ]$ (liên phân số vô hạn). Xét dãy $p_{-2}=0,p_{-1}=1,p_n=x_np_{n-1}+p_{n-2},q_{-2}=1,q_{-1}=0,q_n=x_nq_{n-1}+q_{n-2}$. Khi đó $0<\alpha -\frac{p_{2k}}{q_{2k}}<\frac{1}{q_{2k}^2}\Rightarrow p_{2k}q_{2k}<\alpha q_{2k}^2<p_{2k}q_{2k}+1\Rightarrow \left \lfloor \alpha q_{2k}^2 \right \rfloor=p_{2k}q_{2k}$ (chứng minh chi tiết epsilon 4 p25-36)




#693103 Có tồn tại hình $A$ hay không?

Đã gửi bởi redfox on 15-09-2017 - 21:16 trong Tổ hợp và rời rạc

Bài 1: Không. Ta sẽ chứng minh chu vi $A$ không nhỏ hơn $B$. Ta có thể giả sử $A$ là hình lồi (chu vi bao lồi của $A$ không lớn hơn $A$ và bao lồi của $A$ chứa $B$). Ta chia đường biên của $B$ thành $n$ cung bằng nhau bởi các điểm $B_1,...,B_n$ theo chiều kim đồng hồ, các điểm này tạo thành đa giác lồi $P$. Từ hai điểm $B_i,B_{i+1}$ kẻ hai tia vuông góc với cạnh $B_iB_{i+1}$ và nằm phía ngoài $P$ cắt $A$ tại cung $A_i$. Vì đa giác $P$ và $A$ lồi nên các cung không giao nhau hoặc giao nhau tại một điểm. Ta có chiều dài cung $A_i$ không nhỏ hơn chiều dài cạnh $B_iB_{i+1}$ (đường ngắn nhất nối hai điểm phân biệt là đoạn thẳng, $B_iB_{i+1}$ là khoảng cách giữa hai tia trong cách dựng cung $A_i$ trên). Cho $n$ đến vô cùng ta có đpcm.

Bài 2: Không. Xét cạnh nối hai điểm tùy ý trong $B$ và mặt phẳng qua hai điểm đó, cạnh đó nằm trong tiết diện của mặt phẳng đó với $B$ nên nằm trong $B$, vậy khối $B$ lồi. Sau đó chia bề mặt của $B$ bằng các mặt phẳng cách đều nhau song song với mỗi cặp hai trục tọa độ. Sau đó chiếu như CM trên.

(mình không chắc chắn lắm về lời giải.)




#646867 có bao nhiêu số

Đã gửi bởi redfox on 28-07-2016 - 10:27 trong Tổ hợp và rời rạc

Gọi $F_n$ là số số tự nhiên thỏa mãn đề bài.

Ta sẽ tính $F_{n+2}$

Nếu chữ số đứng thứ hai từ trái sang phải khác $0$ thì có $F_{n+1}$ cách chọn $n+1$ chữ số sau cùng và $8$ cách chọn chữ số đầu tiên. Vậy có $8F_{n+1}$ số.

Nếu chữ số đứng thứ hai bằng $0$ thì có $F_n$ cách chọn $n$ chữ số sau cùng và $9$ cách chọn chữ số đầu tiên. Vậy có $9F_n$ số.

Vậy $F_{n+2}=8F_{n+1}+9F_n$. Ta lại có $F_1=1, F_2=17$. Bằng quy nạp ta chứng minh được $F_n=(4(-1)^n+9^n)/5$.




#646868 Chứng minh đa thức có $n$ nghiệm thực

Đã gửi bởi redfox on 28-07-2016 - 10:33 trong Đa thức

Chứng minh rằng với mỗi số nguyên dương $n$, đa thức $\sum_{k=0}^{n}2^{k(n-k)}x^k$ có đúng $n$ nghiệm thực.




#665431 Chứng minh tập các ước số nguyên tố của dãy vô hạn.

Đã gửi bởi redfox on 22-12-2016 - 07:40 trong Số học

Cho dãy số nguyên dương tăng ngặt $\left ( a_n \right )_{n=1}^\infty$ thỏa $a_n\leq p_n,\forall n\in \mathbb{Z}$, trong đó $p_n$ là số nguyên tố thứ $n$. Chứng minh tập các ước số nguyên tố của dãy vô hạn.




#691418 Chứng minh tập $\mathbb{N^*}\setminus P $ là tậ...

Đã gửi bởi redfox on 24-08-2017 - 17:52 trong Tổ hợp và rời rạc

Theo mình nghĩ ii) là $\forall q\in N,q>1,\exists c\in P,c\not\equiv 0(mod\; q)$ (nếu ii) như trên thử xét $P= \left \{ 4,6,8,...,2k,... \right \}$, và lấy $c= 4q+2$)

Để ý nếu $a,b\in P\Rightarrow ax+by\in P,\forall x,y\in N$ (theo i)). Áp dụng Sylvester ta có nếu $a,b\in P,gcd(a,b)=d\Rightarrow xd\in P,\forall x\in N,x\geq \frac{(a-d)(b-d)}{d^2}$. (*)

Gọi $d=min(gcd(a,b)|a,b\in P)$ theo (*) tồn tại $X$ sao cho $dx\in P,\forall x\in N,x\geq X$.

Giả sử $d>1$. Theo ii) lấy $p=d$ ta có tồn tại $c\in P$ không chia hết cho $d$. Đặt $d'=gcd(c,d)< d$. Lấy $x=cy+1\geq X$ ta có $xd,c\in P,gcd(xd,c)=gcd(cyd+d,c)=d'<d$ (trái với cách chọn $d$. Vậy $d=1$, theo (*) ta dễ có $N/P$ hữu hạn.

(Q.E.D)




#670710 Chứng minh rằng: $S_{k+1}(n+1)=2C_k(n)$

Đã gửi bởi redfox on 08-02-2017 - 12:06 trong Tổ hợp và rời rạc

Cho hai số nguyên dương $n\geq k$, gọi $S_k(n)$ là số chuỗi nhị phân có độ dài $n$ không chứa chuỗi gồm toàn số $0$ hoặc $1$ có độ dài $k$, $C_k(n)$ là số chuỗi nhị phân có độ dài $n$ không chứa chuỗi gồm toàn số $0$.

Chứng minh rằng: $S_{k+1}(n+1)=2C_k(n)$




#651545 Chứng minh rằng: $(\sum_{i=1}^{n}p_i.x_i^2)-(...

Đã gửi bởi redfox on 27-08-2016 - 22:01 trong Các bài toán Đại số khác

Biểu thức trên là tam thức bậc hai đối với $x_i$. Hệ số của $x_i^2$ là $p_i-p_i^2\geq 0$ nên đạt giá trị lớn nhất tại một trong hai đầu mút $a$ hoặc $b$ nên ta chỉ xét trường hợp $x_i$ nhận hai giá trị trên. Đặt $k=\sum_{i\mid x_i=a}p_i, l=\sum_{i\mid x_i=b}p_i$với $k,l\in [0;1], k+l=1$. ta có

$\sum_{i=1}^{n}p_ix_i^2-(\sum_{i=1}^{n}p_ix_i)^2= ka^2+lb^2-(ka+lb)^2=(k+l)(ka^2+lb^2)-(ka+ly)^2=kl(b-a)^2\leq (\frac{k+l}{2})^2(b-a)^2=(\frac{b-a}{2})^2$

(Q.E.D)




#662092 Chứng minh rằng: $\left | S_1 \right |-\left | S_2 \...

Đã gửi bởi redfox on 15-11-2016 - 22:41 trong Tổ hợp và rời rạc

Ta chuyển bài toán về dạng tổng quát hơn: Cho tập $S$ gồm số chẵn phần tử, mỗi tập con được gán một số từ $1$ đến $2^k$ sao cho nếu $A\subset B$ thì số gán cho $B$ lớn hơn. $S_1$ là tập các tập con được gán số $a\leq x\leq b$ có số phần tử chẵn, $S_2$ định nghĩa tương tự.

Nếu hai tập con chẵn trong $S_1$ thỏa $A\subset B$ thì giữa nó có một tập con lẻ trong $S_2$, nếu ta bỏ một trong hai tập con trên và bỏ phần tử lẻ thì hiệu trên không giảm. Vậy ta chỉ cần chứng minh: Nếu tập $I$ gồm các tập con của $S$ không có tập nào là con của tập kia thì $\left | I \right |\leq C_k^{\frac{k}{2}}$.

Gọi $N_i$ là tập các tập con có $i$ phần tử. Ta sẽ chứng minh tồn tại đơn ánh $f_i$ từ $N_i$ đến $N_{i+1}$ thỏa $A\subset f(A)$với $i<\frac{k}{2}$.

Xét đồ thị lưỡng phân $G=(N_i\cup N_{i+1},E)$, nếu $A\subset B$ thì $A$ nối với $B$. Với $X\subset N_i$, số các cạnh nối với $X$ là $E=(k-i)\left | X \right |$, bậc của các phần tử của $N_{i+1}$ là $i$ nên $E$ không lớn hơn $i\left | G(X) \right |$ mà $k-i>i$ nên $G(X)\geq X$, áp dụng định lí Hall ta có điều phải chứng minh.

Tương tự tồn tại đơn ánh $f_i$ từ $N_i$ đến $N_{i-1}$ với $i>\frac{k}{2}$ và $f(A)\subset A$. Sử dụng các đơn ánh, ta đưa các tập con của $I$ về các tập con có $\frac{k}{2}$ phần tử. Vì không có tập nào là con của tập kia nên các tập con sau khi chuyển đổi phân biệt. Vậy $\left | I \right |\leq\left | N_{\frac{k}{2}} \right |=C_k^{\frac{k}{2}}$.

(Q.E.D)

Có cách nào đơn giản hơn không?




#652346 Chứng minh rằng vị trí của O là không đổi sau khi một số hữu hạn bước.

Đã gửi bởi redfox on 02-09-2016 - 07:51 trong Tổ hợp và rời rạc

Gọi các tâm hình tròn theo thứ tự di chuyển là $O_1;O_2;...;O_k;...$. Gọi: 

-Số điểm nằm trong $(O_k;r)$, tổng bình phương khoảng cách từ $O_k, O_{k+1}$ đến các điểm đó lần lượt là $s_k, S_k, S'_k$.

-Số điểm nằm trong $(O_k;r)$ và nằm ngoài $(O_{k+1};r)$, tổng bình phương khoảng cách từ $O_{k+1}$ đến các điểm đó lần lượt là $a_k, A_k$.

-Số điểm nằm ngoài $(O_k;r)$ và nằm trong $(O_{k+1};r)$, tổng bình phương khoảng cách từ $O_{k+1}$ đến các điểm đó lần lượt là $b_k; B_k$.

Ta có:

-$s_k+b_k=s_{k+1}+a_k$.

-$S'_k+B_k=S_{k+1}+A_k$.

-Dựa vào vị trí tương đối của các điểm đối với $(O_{k+1};r)$, ta có $A_k\geq a_kr^2, B_k\leq b_kr^2$.

-Áp dụng Lagrange, ta dễ có $S'_k\leq S_k$.

Từ đây ta có $S_k-s_kr^2\geq S_{k+1}-s_{k+1}r^2$, dùng đơn biến ta dễ có với $k$ đủ lớn, $O_k\equiv O_{k+1}\equiv O_{k+2}\equiv ...$ hay vị trí của $O$ không đổi.

(Q.E.D)




#686021 Chứng minh rằng tồn tại nửa hình tròn chứa đủ các số 1;2;3...;n.

Đã gửi bởi redfox on 30-06-2017 - 16:07 trong Tổ hợp và rời rạc

Bỏ qua TH hiển nhiên tồn tại nửa đường tròn chứa toàn quạt đỏ (hoặc quạt xanh).

Xét $2n$ nửa đường tròn $H_1,...H_{2n}$, $H_{k+1}$ được tạo bằng cách quay $H_k$ theo tâm đường tròn góc $\frac{\pi }{n}$ theo chiều kim đồng hồ. Gọi $Đ_k,X_k$ lần lượt là các số được ghi trên quạt đỏ và quạt xanh đầu tiên trên $H_k$ (thứ tự các quạt theo chiều kim đồng hồ).

Bổ đề: Nếu $d_k=Đ_k-X_k\equiv 1(mod\; n)$ thì quạt $H_k$ chứa đủ các số $1,...,n$

Giả sử $H_k$ chứa $x$ quạt đỏ, $n-x$ quạt xanh. Các số được ghi trên các quạt đỏ đồng dư theo mod $n$ với các số $Đ_k+i=X_k+i+1,0\leq i\leq x-1$, các số được ghi trên các quạt xanh đồng dư theo mod $n$ với các số $X_k-j,0\leq j\leq n-x-1$. Dễ thấy các số được ghi trên các quạt đỏ (quạt xanh) phân biệt. Giả sử tồn tại một số được ghi trên một quạt đỏ và một quạt xanh nào đó suy ra tồn tại $0\leq i\leq x-1,0\leq j\leq n-x-1$ sao cho $X_k+i+1\equiv X_k-j(mod\; n)\Rightarrow i+j+1\equiv 0(mod\; n)$ (vô lý vì $0<i+j+1<n$). Vậy các số ghi trên $H_k$ lập thành hệ thặng dư đầy đủ mod $n$ nên chứa đủ các số $1,...,n$.

Ta xét hai trường hợp: (coi $d_{2n+1}=d_1$)

TH1: Quạt đầu tiên trên $H_k$ màu đỏ, khi đó $Đ_{k+1}\equiv Đ_k+1(mod\; n),X_{k+1}=X_k\Rightarrow d_{k+1}\equiv d_k+1(mod\; n)$

TH2: Quạt đầu tiên trên $H_k$ màu xanh, khi đó $X_{k+1}\equiv X_k-1(mod\; n),Đ_{k+1}=Đ_k\Rightarrow d_{k+1}\equiv d_k+1(mod\; n)$

Vậy $d_{k+1}\equiv d_k+1,\forall 1\leq k\leq 2n$. Dễ thấy $d_1,...,d_n$ lập thành hệ thặng dư đầy đủ mod $n$ nên tồn tại $d_k\equiv 1(mod\; n)$, nửa đường tròn $H_k$ thỏa mãn đề bài.

(Q.E.D)

Có đúng $2$ nửa đường tròn thỏa mãn.




#693653 Chứng minh rằng sau một số lần thực hiện quy tắc thì số 1 xuất hiện ở vị trí...

Đã gửi bởi redfox on 24-09-2017 - 19:35 trong Tổ hợp và rời rạc

(cách trên trừu tượng và khó hiểu quá!)

Ta sẽ chứng minh kết quả mạnh hơn: Các số nguyên dương phân biệt $2\leq a_1<...<a_k\leq n$ được sắp xếp trên hàng có $n$ ô và đổi chỗ như trên nếu ô đầu tiên là số, ô trống sẽ xuất hiện ở ô đầu tiên lúc nào đó (bài trên coi $1$ là ô trống). Ta quy nạp mạnh theo $n$:

$n=1$: trên hàng là ô trống (đúng)

Giả sử với mọi $m<n$ đúng, xét bảng có $n$ ô như trên:

Nếu $a_k< n$ thì $a_k$ ô đầu tiên có ít nhất một ô trống và cách đổi chỗ như trên chỉ ảnh hưởng đến $a_k$ ô đầu tiên. Vì $a_k$ đúng, ô trống sẽ xuất hiện ở ô đầu tiên lúc nào đó nên $n$ đúng

Nếu $a_k=n$, ta coi $n$ là ô trống. Khi đó lập luận tương tự như trên ô trống sẽ xuất hiện ở ô đầu tiên lúc nào đó. Nếu ô trống đó không phải là $n$ thì $n$ đúng. Nếu ô trống đó là $n$, cách đổi chỗ sẽ đưa $n$ xuống cuối. Cách đổi chỗ như trên sau đó chỉ ảnh hưởng đến $n-1$ ô đầu tiên và $n-1$ ô đầu tiên có ô trống. Vì $n-1$ đúng, ô trống sẽ xuất hiện ở ô đầu tiên lúc nào đó nên $n$ đúng.

(Q.E.D)




#655583 Chứng minh rằng $OP\perp AQ$.

Đã gửi bởi redfox on 26-09-2016 - 08:29 trong Hình học

Cho tam giác $ABC$ ($AB\neq AC$) nội tiếp đường tròn $(O)$, trung tuyến $AM$.

Gọi $D$ là hình chiếu của $O$ lên $AM$, tiếp tuyến của $(O)$ kẻ từ $A$ cắt đường thẳng $BC$ tại S.

Một đường thẳng đi qua $S$ cắt đường thẳng $AM$ và đường thẳng qua $D$ song song với $BC$ lần lượt tại $P$ và $Q$.

Chứng minh rằng $OP\perp AQ$.




#684078 Chứng minh rằng $m|ak!$.

Đã gửi bởi redfox on 11-06-2017 - 14:28 trong Đa thức

Cho đa thức $P\left ( x \right )\in \mathbb{Z}\left [ x \right ]$ có bậc là $k$, hệ số cao nhất là $a$ và số nguyên dương $m$.

a) Biết rằng $m|P(x),\forall x\in \mathbb{Z}$. Chứng minh rằng $m|ak!$.

b) Cho $m|ak!$. Chứng minh tồn tại đa thức $P\left ( x \right )\in \mathbb{Z}\left [ x \right ]$ có bậc là $k$, hệ số cao nhất là $a$ thỏa mãn $m|P(x),\forall x\in \mathbb{Z}$.




#645557 Chứng minh $N> M$

Đã gửi bởi redfox on 19-07-2016 - 17:39 trong Số học

Gọi $N$ là số nghiệm nguyên của phương trình $x^2-y^2=z^3-t^3$, với điều kiện $0\leq x, y, z, t\leq 10^6$, và $M$ là số nghiệm nguyên của phương trình $x^2-y^2=z^3-t^3+1$, với điều kiện tương tự. Chứng minh $N> M$.

(IMO Shortlist 1979)




#656479 Chứng minh $F,O,X$ thẳng hàng

Đã gửi bởi redfox on 02-10-2016 - 20:53 trong Hình học

Cho hình chữ nhật $ABCD$ nội tiếp đường tròn $(O)$, điểm $E$ nằm trên $(O)$. Đường thẳng $CE$ cắt $AB$ tại $F$, đường thẳng $DE$ cắt tiếp tuyến của $(O)$ kẻ từ $A$ tại $X$. Chứng minh $F,O,X$ thẳng hàng.(không xài pascal)



#656201 Chứng minh $B, C, E$ thẳng hàng.

Đã gửi bởi redfox on 01-10-2016 - 10:23 trong Hình học

Cho tam giác $ABC$, tâm đường tròn nội tiếp $I$. Điểm $D$ thỏa mãn tứ giác $BICD$ là tứ giác điều hòa. Kẻ đường kính $IE$ của đường tròn ngoại tiếp tam giác $ADI$. Chứng minh $B, C, E$ thẳng hàng.




#694593 chứng minh $\chi (G)\leq \Delta(G)$

Đã gửi bởi redfox on 11-10-2017 - 19:50 trong Tổ hợp và rời rạc

http://myweb.facstaf...kara/brooks.pdf brook's theorem nhé.




#658636 Chứng minh rằng tồn tại $f(A)=H \setminus B,g(B)= H \setmi...

Đã gửi bởi redfox on 20-10-2016 - 22:28 trong Các dạng toán khác

Ta có các bổ đề sau

Bổ đề 1: $A\subset B\Rightarrow A\cap C\subset B\cap C, A\setminus C\subset B\setminus C$.

Bổ để 2: $f(A\cap B)\subset f(A)\cap f(B)$.

Bổ đề 3: Nếu $f$ tăng trên $\rho (H)$ thì $f$ luôn có một điểm bất động.

Chứng minh: Ta chứng minh bằng quy nạp với số phần tử của $H$

$\left | H \right |=0$, hiển nhiên.

Giả sử với $\left | H \right |\leq n$ bài toán đúng. Xét tập $H'=H\cup \left \{ a \right \},H\cap \left \{ a \right \}=\varnothing ,\left | H \right |=n$. Xét hai hàm trên $\rho (H)$: $g(A)=f(A)\setminus \left \{ a \right \},h(A)=f(A\cup \left \{ a \right \})\setminus \left \{ a \right \}$. Theo bổ đề 1, hai hàm này tăng, do vậy theo giả thiết quy nạp tồn tại hai tập $A,B\subset H$ sao cho $g(A)=A, h(B)=B$.

Nếu $f(B\cup \left \{ a \right \})=B\cup \left \{ a \right \}$, bài toán đúng với $n+1$.

Nếu $f(B\cup \left \{ a \right \})=B$, xét tập $A\cap B\setminus H$, theo bổ đề 2 ta có $\forall X\subset A\cap B, f(X)\subset A\cap B$, do vậy theo giả thiết quy nạp bài toán đúng với $n+1$.

Bổ đề $4$: $A\subset B\Leftrightarrow C\setminus B\subset C\setminus A$.

Ta quay lại bài toán. Xét hàm $h(X)=f(H\setminus g(H\setminus X))$. Theo bổ đề 4 ta có hàm $h$ tăng. Theo bổ đề 3 ta có $h$ tồn tại điểm bất động $X$. Dễ thấy hai tập $H\setminus g(H\setminus X) ,H\setminus X$ thỏa mãn đề bài.

(Q.E.D)

Bài này lạ quá. Anh lấy ở đâu vậy.




#658791 Chứng minh rằng tồn tại $f(A)=H \setminus B,g(B)= H \setmi...

Đã gửi bởi redfox on 22-10-2016 - 17:25 trong Các dạng toán khác

Bổ đề: Nếu $f$ tăng trên $\rho (H)$ thì $f$ tồn tại một điểm bất động

Xét dãy tập hợp $X_0=H, X_{n+1}=f(X_n)$. Ta sẽ chứng minh bằng quy nạp $X_{n+1} \subset X_n$.

Với $n=0$, ta có $f(H)\subset H$ vì $f(H)$ thuộc $\rho (H)$ nên là tập con của $H$.

Giả sử $X_{n+1}\subset X_n$, theo định nghĩa hàm tăng, $f(X_{n+1})\subset f(X_n)$ hay $X_{n+2}\subset X_{n+1}$

Vậy $X_{n}$ dần tiến về tập $X\subset H$ (đoạn này thấy không ổn nhưng có vẻ đúng với tập hữu hạn). Ta có $f(X)=X$

Rồi làm như phần chứng minh trên.




#658753 Chứng minh rằng tồn tại $f(A)=H \setminus B,g(B)= H \setmi...

Đã gửi bởi redfox on 22-10-2016 - 09:22 trong Các dạng toán khác

$X=\lim_{n\rightarrow \infty }f^n(H)$




#658657 Chứng minh rằng tồn tại $f(A)=H \setminus B,g(B)= H \setmi...

Đã gửi bởi redfox on 21-10-2016 - 10:50 trong Các dạng toán khác

Không liên quan là sao ạ. Em chứng minh bổ đề 3 với tập không đếm được mà.



#658765 Chứng minh rằng tồn tại $f(A)=H \setminus B,g(B)= H \setmi...

Đã gửi bởi redfox on 22-10-2016 - 11:36 trong Các dạng toán khác

Ta có $f(H)\subset H$ (hiển nhiên), bằng quy nạp $f^{n+1}(H)\subset f^n(H)$. Vậy $f^n(H)$ có giới hạn thỏa $f(X)=X$.

Không biết xài mấy từ này có ổn không.




#652998 C/m:$\forall r\in Q+$ thì có duy nhất n là số dương sao c...

Đã gửi bởi redfox on 06-09-2016 - 14:09 trong Dãy số - Giới hạn

Với $a,b\in \mathbb{Z}^+;gcd(a,b)=1$, viết $\frac{a}{b}=k_m+\frac{1}{k_{m-1}+\frac{1}{...+\frac{1}{k_2+\frac{1}{k_1}}}}$ trong đó $m,k_1,k_2,...,k_{m-1}\in \mathbb{Z}^+;k_m\in \mathbb{N}$.

Khi đó $n=1\underbrace{0..0}_{k_1-1}1\underbrace{0...0}_{k_2-1}1...1\underbrace{0...0}_{k_{m-1}-1}1\underbrace{0...0}_{k_m}$ (viết dưới dạng nhị phân) thoả mãn $x_n=\frac{a}{b}$.

Với mỗi phân số $\frac{a}{b}$, xác định duy nhất các số $m,k_1,k_2,...,k_m$ và xác định duy nhất $n$.

Với mỗi $n\in \mathbb{Z}$, xác định duy nhất các số $m,k_1,k_2,...,k_m$ và xác định duy nhất phân số $\frac{a}{b}$

Vậy đây là song ánh, dễ suy ra bài toán.

(Q.E.D)




#684338 C/m: $max (n)\leq [k!e]-1$

Đã gửi bởi redfox on 13-06-2017 - 14:21 trong Tổ hợp và rời rạc

Bổ đề: $\left \lceil \frac{\left \lfloor k!e \right \rfloor}{k} \right \rceil=\left \lfloor \left ( k-1 \right )!e \right \rfloor+1$ (có thể chứng minh nhờ công thức: $e=\sum_{k=1}^{\infty }\frac{1}{k!}$).

Ta định nghĩa: $A-B=\left \{ a-b|a\in A,b\in B,a>b \right \}$. Để ý rằng $A\subset C,B\subset D\Rightarrow A-B\subset C-D$.

Ta sẽ chứng minh nếu tập $N\subset \mathbb{Z}^+$ có tập con $M$ thỏa mãn $\left | M \right |\geq \left \lfloor k!e \right \rfloor,M-M\subset N$ thì không tồn tại cách phân hoạch thỏa mãn đề. Ta chứng minh bằng quy nạp theo $k$.

Với $k=1$, dễ dàng chứng minh bài toán đúng.

Giả sử $k-1$ đúng, xét phân hoạch $N$ thành $k$ tập hợp $A_1,A_2,...,A_k$. Theo Dirichlet, tồn tại $A_i$ sao cho $\left | A_i\cap M \right |\geq \left \lceil \frac{\left | M \right |}{k} \right \rceil\geq \left \lfloor \left ( k-1 \right )!e \right \rfloor+1$. Gọi $x$ là phần tử lớn nhất của $A_i\cap M$. Đặt $M'=\left \{ x \right \}-A_i\cap M$, ta có $\left | M' \right |\geq \left \lfloor \left ( k-1 \right )!e \right \rfloor$. Dễ thấy $M',M'-M'\subset A_i\cap M-A_i\cap M\subset M-M\subset N;M',M'-M'\subset A_i-A_i$

TH1: $M'\cap A_i\neq \varnothing$ hoặc $\left ( M'-M' \right )\cap A_i\neq \varnothing$, ta chọn được ba số trong $A_i$, một số bằng tổng hai số còn lại nên cách phân hoạch này không thỏa mãn.

TH2: $M',M'-M'\subset N'=N/A_i$, áp dụng bài toán với $k-1,N',M'$ với cách phân hoạch $N'$ thành $k-1$ tập hợp $A_1,A_2,...,A_k$ (trừ $A_i$), cách phân hoạch này không thỏa mãn với $N'$ nên cách phân hoạch $N$ thành $k$ tập hợp $A_1,A_2,...,A_k$ như trên cũng không thỏa mãn đề.

Áp dụng bài toán với $N=M=\left \{ 1,2,..,n \right \},n\geq \left \lfloor k!e \right \rfloor$ ta thấy không tồn tại cách phân hoạch thỏa mãn, vậy $n\leq \left \lfloor k!e \right \rfloor-1$.

(Q.E.D)